subject
Mathematics, 06.07.2021 23:10 bbgrl3018

Describe the motion of a particle with position (x, y) as t varies in the given interval. (For each answer, enter an ordered pair of the form x, y.) x = 1 + sin(t), y = 3 + 2 cos(t), π/2 ≤ t ≤ 2π

ansver
Answers: 2

Another question on Mathematics

question
Mathematics, 20.06.2019 18:02
Does anyone know this? the andwer choices are: 79, 85, 101, and 135 degrees!
Answers: 2
question
Mathematics, 21.06.2019 17:00
Mary beth used the mapping rule to find the coordinates of a point that had been rotated 90° counterclockwise around the origin. examine the steps to determine whether she made an error. m (3, –6) is rotated 90° counterclockwise. (x, y) → (–y, x) 1. switch the x- and y-coordinates: (6, –3) 2. multiply the new x-coordinate by –1: (6(–1), –3) 3. simplify: (–6, –3) .
Answers: 1
question
Mathematics, 21.06.2019 19:50
Given that abcd is a rhombus, what is the value of x?
Answers: 3
question
Mathematics, 21.06.2019 20:00
Simplify 24÷(-2)(3)+7 a. -29 b. 3 c. 11
Answers: 1
You know the right answer?
Describe the motion of a particle with position (x, y) as t varies in the given interval. (For each...
Questions
question
Mathematics, 10.12.2020 17:30
question
Mathematics, 10.12.2020 17:30
question
Mathematics, 10.12.2020 17:30
question
Mathematics, 10.12.2020 17:30
question
Mathematics, 10.12.2020 17:30
Questions on the website: 13722367